From mboxrd@z Thu Jan 1 00:00:00 1970 X-Msuck: nntp://news.gmane.io/gmane.science.mathematics.categories/1469 Path: news.gmane.org!not-for-mail From: maxkanov@math.upenn.edu (Max Kanovitch) Newsgroups: gmane.science.mathematics.categories Subject: Re: Re: stupid question? Date: Wed, 29 Mar 2000 18:13:53 -0500 (EST) Message-ID: <200003292313.SAA14643@hans.math.upenn.edu> NNTP-Posting-Host: main.gmane.org X-Trace: ger.gmane.org 1241017851 31379 80.91.229.2 (29 Apr 2009 15:10:51 GMT) X-Complaints-To: usenet@ger.gmane.org NNTP-Posting-Date: Wed, 29 Apr 2009 15:10:51 +0000 (UTC) To: categories@mta.ca Original-X-From: rrosebru@mta.ca Wed Mar 29 19:21:46 2000 -0400 Original-Received: (from Majordom@localhost) by mailserv.mta.ca (8.9.3/8.9.3) id TAA18092 for categories-list; Wed, 29 Mar 2000 19:20:01 -0400 (AST) X-Authentication-Warning: mailserv.mta.ca: Majordom set sender to cat-dist@mta.ca using -f Original-Sender: cat-dist@mta.ca Precedence: bulk Original-Lines: 28 Xref: news.gmane.org gmane.science.mathematics.categories:1469 Archived-At: Dear M.M. Mawanda, > >I have been asked the following question: Is it true that any function > >defined in a real number closed interval [a,b] (there is not a hypothesis > >of continuity) is bounded in an open subinterval (c,d) of [a,b]? The real fun is about a function f such that f is unbounded in any open interval (c,d), and in addition to that: f(x+y) = f(x)+f(y). > Date: Wed, 29 Mar 2000 15:23:16 -0500 (EST) > From: Peter Freyd > Subject: categories: Re: stupid question? > > M.M. Mawanda asks: > > >I have been asked the following question: Is it true that any function > >defined in a real number closed interval [a,b] (there is not a hypothesis > >of continuity) is bounded in an open subinterval (c,d) of [a,b]? My > >spontaneous was NO. Unfortunately I cannot find a counter-example to > >disapproved my answer. Can someone help. > > No it is not true. For example, the function defined by: > > f(x) = if x is irrational then 0 else > if x = p/q where p and q are co-prime then q. >